Thread Rating:
  • 0 Vote(s) - 0 Average
  • 1
  • 2
  • 3
  • 4
  • 5
nbme 2, block 1 - rizowana
#31
11-J
12-HH
Reply
#32
very confusing q

13. Is it renal artery stenosis?

14. A, hyperaldosteronism?
Reply
#33
13-DD
14-AA
Reply
#34
11j
12h
13 d
14 a
Reply
#35
15.) A 24-year-old man is hospitalized for treatment of a posterior dislocation of the
right knee sustained in a motorcycle collision. Six hours after closed reduction,
previously present distal pulses in his foot are absent, but the foot has remained
warm. Which of the following is the most appropriate next step in management?


A) Elevation of the limb and observation for 24hours

B) Nitroprusside therapy

C) Sympathetic block

D) Femoral arteriography

E) Embolectomy with a Fogarty catheter through aproximal arteriotomy

D) Inflammation of the origin of the plantar fascia





16.) A healthy 4-year-old girl is brought for a well-child examination. A grade 2/6
systolic ejection murmur is heard along the upper left sternal border. S2 is widely
split and does not vary with respiration. A soft mid-diastolic murmur is heard along
the lower left sternal border. Examination shows no other abnormalities. Which of
the following is the most likely diagnosis?


A) Aortic stenosis

B) Atrial septal defect

C) Coarctation of the aorta

D) Mitral valve prolapse

E) Patent ductus arteriosus

F) Pulmonary stenosis

G) Tetralogy of Fallot

H) Transposition of the great arteries

I) Ventricular septal defect

J) Normal heart
Reply
#36
15. D?

16. B?
Reply
#37
16-BB
Reply
#38
18.) A previously healthy 57-year-old woman comes to the physician because of three
episodes of blurred vision in the right eye over the past 3 weeks; each episode
lasts approximately 5minutes. Retinal examination shows a small refractile body at
The bifurcation of a retinal artery. The remainder of the examination shows no
abnormalities. Which of the following is the most appropriate next step in
diagnosis?

A) Cerebral angiography

B) Echocardiography

C) Electroencephalography

D) Duplex scan of the carotid arteries

E) MRI of the brain


19.) A 67-year-old woman comes to the physician for her first influenza virus
vaccination. She has a history of untreated hypertension. Her blood pressure is
160/100 mm Hg, and pulse is 100/min. Shortly after administration of the
influenza virus vaccine, she develops shortness of breath, hives, and angioedema.
Which of the following is most likely to have prevented this reaction?


A) Inquiry about an egg allergy

B) Heterophile agglutination test

C) Skin test with histamine reagent

D) ß-Adrenergic blocking agent therapy

E) Amantadine therapy

F) Insulin therapy

G) Rimantadine therapy


Reply
#39
18. A or C?

19. A
Reply
#40
18- EXAMINATION ..NO ABNORMALITIES+COMMON SOURCE OF EMBOLI=DD
Reply
« Next Oldest | Next Newest »


Forum Jump: